More Solitary Passages Questions - - Question 27

The main purpose of the passage is to

Shiyi-Zhang January 23, 2019

Why is E correct?

Why is E correct?

Replies
Create a free account to read and take part in forum discussions.

Already have an account? log in

Shiyi-Zhang January 30, 2019

Could someone explain this question to me?

Ravi January 31, 2019

@Shiyi-Zhang,

Happy to help! This question is asking us to identify the main point
of the passage. On these types of questions, it's always helpful to
try to anticipate the main point of the passage immediately after
reading it because this question is almost always asked.

In the passage, we're presented with 3 formulations of the "classical"
social psychological theory of social movement. The passage then
shifts to criticizing these formulations (first by political
theorists, and then empirically by the author).

Now that we've identified the main gist of the passage, let's look at
the answers.

(A) is incorrect because it fails to discuss how the passage mentions
that the social psychological theories have problems.

(B) is incorrect because the author doesn't present a new model;
rather, she's focused on tearing down old ones.

(C) is incorrect because there is no shift in theoretical debate that
is mentioned by the author.

(D) is incorrect because although it's true, it's confined to the
second paragraph and is the main purpose of that paragraph. However,
(D) is too limited in scope and doesn't factor in the other things
that this passage is about. The rest of the passage tears down the
social psychological theories.

(E) is correct because it says that the main purpose of the passage is
to discuss the reasoning behind and shortcomings of certain social
psychological theories. This matches up well with the what we
anticipated the main point answer saying, so it's our correct answer
choice. The reasoning of the certain social psychological theories is
contained in the second and third paragraphs, and the shortcomings of
these theories are described in the third and fourth paragraphs. This
is the correct choice.

Does this make sense? Let us know if you have any more questions!

JeremyG March 29, 2019

I am not sure how you can reconcile the claim that “there is no shift in theoretical debate that is mentioned by the author” against this quote from lines 5-8: “New theories of social movements are being discussed, not just among psychologists, but also among political theorists.”

I was between choices C and E but ultimately decided on the former on the basis of that quote. E seemed wrong because it failed to discuss the political theorists' point of view. I am still not entirely sure why C is a weaker answer than E. Please advise.

Jacob-R March 29, 2019

Hi @JeremyG - I just see a blank question. Can you repost please?

JeremyG April 9, 2019

Hi Jacob, reposting:


I am not sure how you can reconcile the claim that “there is no shift in theoretical debate that is mentioned by the author” against this quote from lines 5-8: “New theories of social movements are being discussed, not just among psychologists, but also among political theorists.”

I was between choices C and E but ultimately decided on the former on the basis of that quote. E seemed wrong because it failed to discuss the political theorists' point of view. I am still not entirely sure why C is a weaker answer than E. Please advise.

Katherine April 11, 2019

Hi @JeremyG - You question posted as another blank. There may be a technical issue. Please direct any support related issues to our support staff by tapping "support" from the left menu or by calling 855.483.7862 ext. 2 Monday-Friday 9am-6pm PT. I hope this helps!